assuming that no one is born on feb. 29 (leap day), how many people should be selected to guarantee that at least 5 were born on the same day, not considering the year?

Answers

Answer 1

365 sets of people born on the same day, with each set containing no more than 5 people.

Assuming everybody was born on a different day, then you could have 365 people where nobody was born on the same day. the 366 person, however, would have to have been born on the same day as somebody else in the group. therefore, the minimum number of people in the group would have to be 366 to guarantee that at least 2 people were born on the same day but you wanted to guarantee that at least 6 people were born on the same day. assuming you had 5 * 365 people together, with every 5 of them being born on the same day.

you would have a total of 365 * 5 = 1825 people, with no more than 5 people being born on the same day.

Add 1 more person and you will be guaranteed that at least 6 people were born on the same day that comes out to be 1826 people.

1825 / 5 = 365 sets of people born on the same day, with each set containing no more than 5 people.

set 1 is the number of people born on day 1.

set 2 is the number of people born on day 2 etc.

that next person had to be added to one of the sets of 5 to make it a set of 6.

Learn more about birthday problems at:

https://brainly.com/question/13493824

#SPJ4


Related Questions

A company manufactures model rockets that require igniters to launch. Once an igniter is used to launch a rocket, the igniter cannot be reused. Sometimes an igniter fails to operate correctly, and the rocket does not launch. The company estimates that the overall failure rate, defined as the percent of all igniters that fail to operate correctly, is 15 percent. A company engineer develops a new igniter, called the super igniter, with the intent of lowering the failure rate. test the performance of the super igniters, the engineer uses the following process. Step 1: One super igniter is selected at random and used in a rocket. Step 2: If the rocket launches, another super igniter is selected at random and used in a rocket. Step 2 is repeated until the process stops. The process stops when a super igniter fails to operate correctly or 32 super igniters have successfully launched rockets, whichever comes first. Assume that super igniter failures are independent a. If the failure rate of the super igniters is 15 percent, what is the probability that the first 30 super igniters selected using the testing process successfully launch rockets? b. Given that the first 30 super igniters successfully launch rockets, what is the probability that the first failure occurs on the thirty-first or the thirty-second super igniter tested if the failure rate of the super igniters is 15 percent? C. Given that the first 30 super igniters successfully launch rockets, is it reasonable to believe that the failure rate of the super igniters is less than 15 percent? Explain.

Answers

0.0076 is  the probability that the first 30 igniters tested do not fail .

What is probability in math?

The area of mathematics known as probability explores potential outcomes of events, along with the likelihoods and distributions of those occurrences.

a)if the failure rate for the super igniters is 15% then the probability that each igniter fails is 0.15,and the probability that it does not fail is 0.85.Therefore the probability that the first 30 igniters tested do not fail is       (0.85)30= 0.0076.

b)given that there are no failures in the first 30 tries,the probabilities occurs on 31st try is 0.15 and the probability that it does not occur on 31st but occurs on 32nd try is (0.85)(0.15)=0.1275.therefore the probability of one or the other is 0.15+0.1275=0.2775.

c)the result of the probability calculation in part(a) provides reason to believ the failure rate of the super igniters is less than 15%.The calculated proability of 0.0076 shows that there is less than a 1% chance that 30 or more igniters in a row would not fail if the failured rate was 15%.This probability is smaller than conventional significance levels such as alpha=0.05 and 0.01 and thus is small enough to make it reasonable to believe that the failure rate of the super igniters is less than 15%.

Learn more about Probability

brainly.com/question/11234923

#SPJ1

this data is normally distributed. what percent of the data is in the shaded region 68% 50% 95% 99.7%

Answers

As per the empirical rule, the percent of data in the shaded region is 68%.

In statistics, the empirical rule states that When you use a standard normal distribution also known as Gaussian Distribution we have to consider the following

=> Around  68% of values fall within one standard deviation of the mean.

=> Around 95% of the values fall within two standard deviations from the mean.

=> Other all of the values are about 99.7% that fall within three standard deviations from the mean.

Therefore, this facts are the 68 95 99.7 rule. And it is also called the Empirical Rule because the rule originally came from observations.

Here we have given the the data is normally distributed. And we need to find the percent of data is in the shaded region.

While we looking into the following diagram and according to empirical rule,68% of the data falls within one standard deviation of the mean.

Because, the center part is the shaded region,

So, it can be calculated as,

=> 34 + 34 = 68%

To know more about Standard deviation here

https://brainly.com/question/16555520

#SPJ4

How do you determine whether the sequence a_n=(2^n+3^n) / (2^n−3^n) converges, if so how do you find the limit?

Answers

You determine the limit by [tex]\lim_{n \to \infty} a_n = -1[/tex].

A limit in mathematics is a point at which a function approaches the output for the specified input values. Calculus and mathematical analysis depend on limits, which are also used to determine integrals, derivatives, and continuity. The concept of limit, which is founded on the idea of closeness, is largely used to give values to some functions at points when none are defined in a way that is consistent with values close by.

Divide the denominator and numerator by 3n as shown.

[tex]\lim_{n \to \infty} a_n = \lim_{n \to \infty} \frac{2^n+3^n}{2^n-3^n\\}[/tex]

[tex]= \lim_{n \to \infty} \frac{(\frac{2}{3})^n+1 }{(\frac{2}{3})^n-1 } \\=\frac{0+1}{0-1} \\=-1[/tex]

To learn more about limits, refer:-

https://brainly.com/question/12211820

#SPJ4

we want to conduct a hypothesis testing about comparing of the 95th percentile of the income of country a and that of country b. we take a sample from each country. can the permutation test be used in this problem?

Answers

Yes, the permutation test is used in this problem to conduct a hypothesis testing about comparing  the 95th percentile of the income of country a and that of country b.

Yes, the permutation test can be used in this problem. The permutation test is a nonparametric test that can be used to compare two or more groups to determine if there is a statistically significant difference between them. In this case, we can use the permutation test to compare the 95th percentile of the incomes of the two countries. The permutation test would involve randomly permuting the data for each country, then calculating the 95th percentile for each permutation. The difference in the 95th percentile between the two countries in the original data can then be compared to the differences in the 95th percentile of the permuted data to see if the difference is statistically significant.

To know more about permutation tests refer to the link  brainly.com/question/3867157

#SPJ4

Which of the following quadrilaterals has four right angles and 4 congruent sides? A. parallelogram B. rectangle C. rhombus D. square

Answers

Answer:Square

Step-by-step explanation:

A square is a quadrilateral with four right angles and four congruent sides. All squares are rectangles and rhombuses.

Answer:

D. square

Step-by-step explanation:

As an explanation, I have attached a picture of the 4 quadrilaterals and their internal angles and sides.

⭐if this response helped you, please mark it the "brainliest"!⭐

water runs into a conical tank at the rate of 9 ft 3/min. the tank stands point down and has a height of 10 ft and a base radius of 5 ft. how fast is the water level rising when the water is 6 ft deep?

Answers

The water level rise when it is 6 feet deep is 0.318 feet/min when a conical tank is filled with water at a pace of 9 feet per minute.

Given that,

A conical tank is filled with water at a pace of 9 feet per minute. The tank is 10 feet tall and has a base radius of 5 feet. It stands pointed down.

We have to find how quickly does the water level rise when it is 6 feet deep.

We know that,

dv/dt= 9 feet³/min

x=5 feet

y=10 feet

And we have x/y=1/2 ----->equation(1)

We know the formula

V=1/3πx²y

Differentiating on both sides

dv/dt=9

1/3πy(2x) dx/dt+π/3(x²)dy/dt

=π/3[2xy dx/dt+x² dy/dt]

From equation(1)

x/y=1/2       or     y=2x

dy/dt= 2dx/dt

Now,

9=π/3[xy dy/dt+x² dy/dt]

9= π/3 dy/dt[xy+x²]

dy/dt=9×3/π×(18+9)

dy/dt=27/π×27

dy/dt= 1/π

dy/dt=0.318 feet/min

Therefore, The water level rise when it is 6 feet deep is 0.318 feet/min when a conical tank is filled with water at a pace of 9 feet per minute.

To learn more about feet visit: https://brainly.com/question/15658113

#SPJ4

Solve for x: 3/4(x-13)=-2(9+x)

Answers

Answer:

x = -3

Step-by-step explanation:

Welcome to Brainly! :]

3/4(x-13)=-2(9+x)

0.75x - 9.75 = -18 - 2x

2.75x = -8.25

x = -3

:]

what can you say about the geometric multiplicity of the eigenvalues of a matrix of the form a = ⎡ ⎣ 010 001 abc ⎤ ⎦ , where a, b, c are arbitrary constants?

Answers

Eigenvalues are a unique collection of scalar values connected to the set of linear equations that are most often found in the matrix equation.

What is meant by eigen value?

Eigenvalues are the unique set of scalar values connected to the set of linear equations most likely found in the matrix equations. Also known as characteristic roots, the eigenvectors are. After applying linear transformations, the vector is non-zero and can only be altered by its scalar factor.

The term "eigenvalue equation" refers to an equation in which the operator multiplies the function by a constant when applied to a function. An eigenvalue is the resultant numerical value, and an eigenfunction is the function itself.

Given:

The given value is

[tex]$$A=\left[\begin{array}{lll}0 & 1 & 0 \\0 & 0 & 1 \\a & b & c\end{array}\right]$$[/tex]

where a, b, c are arbitrary constants.

If [tex]$\lambda$[/tex] is an eigen value of A Then,

[tex]$& E_\lambda={ker}(A-\lambda I) \\[/tex]

Now,

Last two rows are linearly independent.

So, we have

[tex]${dim} E_\lambda=1$[/tex]

Therefore,

[tex]${gemu}(\lambda)=1$$[/tex]

To learn more about eigen value refer to:

https://brainly.com/question/2289152

#SPJ4

What is 100k in Celsius ?

Answers

100k in celsius is -173.15 degrees.

The Celsius scale, commonly known as centigrade, uses a base of 0° for water's freezing point and 100° for water's boiling point. Because of the 100-degree gap between the designated points, the Celsius scale, which was developed in 1742 by the Swedish astronomer Anders Celsius, is occasionally referred to as the centigrade scale.

The Celsius scale is in general use wherever the metric system of units has been adopted, and it is used in scientific work everywhere.

To convert Celsius to kelvin, the formula is:

T(K)=°C+273.15

∴100=°C+273.15

°C=100-273.15 = - 173.15

To know more about celsius visit: brainly.com/question/11524365

#SPJ4

9,15,25, Find the 10th term

Answers

On solving thr provided question, by help of arithmetic progression, we got to know that a10 = 207

what is arithmetic progression?

There are two approaches to define arithmetic progressions (AP):

The difference between any two successive terms in an arithmetic sequence must always be equal, and it is sometimes referred to as a series in which all terms except the first are created by adding a predetermined number to the preceding term.

9,15,25..............

15-9 = 6

25-15 = 10

a4 - 25 = 14 => a4 = 39

a5-39 = 18 => a5 = 57

a6-57 = 22 => a6 79

...

...

...

...

...

a10 - 169 = 38 => a10 = 207

To know more about arithmetic progression visit:
https://brainly.com/question/16947807

#SPJ1

The solution of the equation 2x+4=3x-62x+4=3x−6 is x =

Answers

the answer , answer :45xy

prove Triangle ACE is an isosceles triangle by completing this proof.
Hint: there are 3 parts to this proof!

Answers

An isosceles triangle is a type of triangle which has two of its sides to be equal, viz-a-viz two of its internal angles are equal. The required proof in the given question are stated below:

A triangle is a trigon with just three straight sides, thus three internal angles. Thus, the sum of its internal angles equals [tex]180^{o}[/tex]. Some common examples are. equilateral, isosceles, right angled, scalene, etc.

An isosceles triangle is one which has two of its sides equal, so that the measure of two of its internal angles are also equal.

Therefore, the expected proof in the question are:

              STATEMENT                                             REASON

1. AB = BC = CD = DE = EA        Similar sides of a regular pentagon  

2. AB = DE                                  Congruent of sides of similar triangle

3. BC = CD                                 Congruent of sides of similar triangle

4. <ABC = <EDC                         Congruent angles of similar triangle

5. AC = CE                                 Congruent sides of an isosceles triangle

Learn more about the congruent properties of an isosceles triangle at https://brainly.com/question/1584654

#SPJ1

2-[(2+10(-1)divided by 2)+1]
What is the value on the expression?

Answers

The value of the numerical expression 2 - [(2 + 10 × -1) / 2 + 1] will be 5.

What is the value of the expression?

When the relevant components and basic processes of a numerical method are given values, the expression's result is the result of the computation it depicts.

The acronym PEMDAS stands for Parenthesis, Exponent, Multiplication, Division, Addition, and Subtraction. This approach is used to answer the problem correctly and completely.

The numerical expression is given below.

⇒ 2 - [(2 + 10 × -1) / 2 + 1]

Simplify the expression, then we have

⇒ 2 - [(2 + 10 × -1) / 2 + 1]

⇒ 2 - [(2 - 10) / 2 + 1]

⇒ 2 - [(-8 / 2) + 1]

⇒ 2 - (-4 + 1)

⇒ 2 - (-3)

⇒ 2 + 3

⇒ 5

The value of the numerical expression 2 - [(2 + 10 × -1) / 2 + 1] will be 5.

More about the value of the expression link is given below.

https://brainly.com/question/23671908

#SPJ1

What transformation has changed the parent function f(x) = (.5)x to its new appearance shown in the graph below? exponential graph passing through point 1, 2 and point 2, 1. (1 point) f(x) − 2 f(x + 2) f(x − 2) f(x) + 1

Answers

The transformation that has changed the parent function f(x) = (0.5)^x to the exponential graph passing through points (1,2) and (2,1) is given as follows:

f(x - 2).

How to define the transformation?

The parent function for this problem is given as follows:

f(x) = (0.5)^x.

Hence the points on the graph of the parent function are given as follows:

(-1, 2), as when x = -1, f(x) = (0.5)^(-1) = (1/0.5)^1 = 2.(0, 1), as when x = 0, f(x) = (0.5)^0 = 1.(1, 0.5), as when x = 1, f(x) = (0.5)^1 = 0.5.(2, 0.25), as when x = 2, f(x) = (0.5)² = 0.25.

The points on the transformed function are given as follows:

(1,2) and (2,1).

Meaning that the value assumed at x = -1 is now assumed at x = 1, while the value assumed at x = 0 is not assumed at x = 2, meaning that the function was shifted right two units, and the transformation is defined as follows:

f(x - 2).

More can be learned about transformations at https://brainly.com/question/28792248

#SPJ1

4700 people attended a football game. If 2% of the people who attended were
teenagers, how many teenagers attended the game?
Insert the values given in the problem then scale up or
down to find the missing value.
attendees
percent
100

Answers

The number of teenagers in the game are 94.

What is percentage?

Percentage is defined as a given part or amount in every hundred. It is a fraction with 100 as the denominator and is represented by the symbol "%".

Given that, 4700 people attended a football game.

Here, 2% of the people who attended were teenagers

So, the number of teenagers are

2% of 4700

= 0.02×4700

= 94

Therefore, there are 94 teenagers attended the game.

To learn more about the percentage visit:

brainly.com/question/24159063.

#SPJ1

What is the value of x?

Answers

The value of x for the given expressions is 3.

What is equilateral triangle?

Triangles are classified into three different types based on their sides. An isosceles triangle, a scalene triangle, and an equilateral triangle. Equilateral triangles are different from isosceles and scalene triangles.

An equilateral triangle is a triangle with all three equal sides and equal angles. An equilateral triangle is also called an equilateral triangle because each angle has a value of 60 degrees. Equilateral triangles are considered regular polygons or equilateral triangles because they have equal angles and equal sides.

In ∆ ABC

∠A= ∠B=∠C

Triangle is equilateral triangle.

All the triangle side will be equal

6x-3=3x+6=5x

=6x-3 =3x+6

=3x=g

X=3

Thus, X = 3

To know more about equilateral triangle, click-

https://brainly.com/question/10147636

#SPJ1

What is the set of numbers to which √10.24 belong?

Answers

√10.24 belongs to the set of rational numbers.

Here we are given the number √10.24.

Now first we see that √10.24

= 3.2

Now as we can see that there is a number other than 0 after the decimal point here. Hence the above number cannot be classified as a natural number, a whole number, or an integer.

However, on converting 3.2 into the fractional form, we see that

3.2 = 32/10

reducing this to smallest form we get

16/5

Here we could easily represent 3.2 in the form of p/q where q is not equal to 0.

Hence p/q is a rational number.

To learn more about Rational Numbers visit

https://brainly.com/question/24398433

#SPJ4

Evaluate 9.1p + 8.9r when p = 6 and r =7.

Answers

The solution to the expression 9.1p + 8.9r is 116.9 when p = 6 and r = 7

How to evaluate the mathematical expression?

From the question, the mathematical expression is given as

9.1p + 8.9r

The values of the variables are given as

p = 6 and r = 7

Substitute p = 6 and r = 7 in the given expression 9.1p + 8.9r

So, we have the following representation

9.1p + 8.9r = 9.1 * 6 + 8.9 * 7

Evaluate the products in the above equation

9.1p + 8.9r = 54.6 + 62.3

Evaluate the sum in the above equation

9.1p + 8.9r = 116.9

Hence, the value is 116.9

Read more about expressions at

brainly.com/question/4344214

#SPJ1

jacob, a researcher, asks the participants of his study to transfer data from 12 spreadsheets to 12 different documents. he observes that the participants gain more efficiency in transferring the data with each new spreadsheet. this scenario exemplifies a(n)

Answers

The scenario exemplifies a(n) that effect is practice effect

In experiments utilising within-subjects designs, the practise effect is especially problematic since participants' performance on the study variable may increase just by engaging in the activity again, independent of any study manipulations the researcher may have imposed.

When a test is retaken, the practise effects have an impact on the results. A practise effect, for instance, can result from taking numerous practise SAT exams; practise can raise your overall score.

Counter balancing is one strategy for limiting the effects of practise. It is possible that the trials will use randomization or will use every feasible order for administering treatments to subjects.

According to the question,

Jacob observes that the participants gain more efficiency in transferring the data with each new spreadsheet. this scenario exemplifies a(n) that this is Practice effect

To know more about Practice effect here

https://brainly.com/question/28616617

#SPJ4

I NEED HELP!!!!!!!!!!!!

Answers

Step-by-step explanation:

the surface area is the combination of

the base (square) area

the 4 side (triangle) areas.

the base area is : 10×10 = 100 in²

the area of a triangle is

baseline × height / 2

we know the baseline of each of the 4 triangles (10 in) and we know the height (14 in).

one triangle area is

10×14/2 = 70 in²

we have 4 of these triangles (each side of the pyramid) :

4×70 = 280 in²

the total surface area of the pyramid is

100 + 280 = 380 in²

Every fall, Maya has to rake the leaves in her backyard. She wants to know the area of her backyard knows what space that she has to rake Three of the coordinates on the grid that can be used to represent her backyard are (8.5, 4), (8.5, 8) and (-2, 8). 1. Plot the fourth coordinate of the rectangle. What is the ordered pair? ​

Answers

Answer:

(2,4)

Step-by-step explanation:

which graph shows a polynomial function with a positive leading coefficient?

Answers

The graph shows a polynomial function with a positive leading coefficient B.

What is meant by polynomial function with a positive leading coefficient?

The function will reach its maximum value of + in the case of a positive leading coefficient and its minimum value of - in the case of a negative leading coefficient. This indicates that even degree polynomials with positive leading coefficients have a range of  [ymin, ∞), where ymin stands for the global minimum that the function reaches.

When a variable in an equation like the quadratic equation, cubic equation, etc. has only non-negative integer powers or only positive integer exponents, the function is said to be polynomial.

Leading coefficients are the values listed before the variable with the biggest exponent. They can be positive, negative, real, imaginary, whole, fractional, or decimal, much like conventional coefficients, and they can also be positive, negative, real, or imaginary.

Therefore, the correct answer is option B.

To learn more about polynomial function refer to:

https://brainly.com/question/2833285

#SPJ4

HELP MEEEEEEEEEEEEEEEE PLEASEEEEEEEEEEEE!!!!!!!!!!!!!!!!!!!!!!!!!!!!!

Answers

Answer:

t ≈ 11.7

Step-by-step explanation:

                  r

A = P (1 + -------)^(n)(t)

                  n

A = 5000

P = 2500

r = 0.06

n = 2

t = ?

                              0.06

5000 = 2500 (1 + ----------)^(2)(t)

                                  2

                              0.06

5000 = 2500 (1 + ----------)^(2)(t)

÷2500 ÷2500           2

             0.06

2 = (1 + ----------)^(2)(t)

                2

            log 2

------------------------ =  2t

             0.06

log (1 + ----------)

                2

            log 2

(1/2) ------------------ = 2t (1/2)

                    0.06

      log (1 + ----------)

                      2

11.72488612 = t

11.7 ≈ t

I hope this helps!

A map has a ratio of 1:4000. Find the actual distance, in metres, which is represented by 6.4 cm on the map.​
Show working

Answers

The scale of a map is the relationship between the distance on the map and the actual distance. In this case, the scale of the map is 1:4000, which means that 1 unit on the map represents 4000 units in the actual distance. To find the actual distance that is represented by 6.4 cm on the map, we need to convert the distance on the map to the same unit as the actual distance. Since 1 cm is equal to 0.01 m, 6.4 cm is equal to 6.4 * 0.01 = 0.064 m.

Now we can use the scale of the map to convert the distance on the map to the actual distance. We can set up the following proportion:

0.064 m / 1 = x / 4000

Where x is the actual distance in metres. Solving for x, we get:

x = 0.064 m * 4000 = 256 m

Therefore, the actual distance represented by 6.4 cm on the map is 256 m.

pq²(p² - 3pq - 4q²) answer fast pls

Answers

This is the answer and the “^” means it’s raise to a power: P^3 Q^2 -3p^2 q^3 -4pq^4

question 4 you are working with the following selection of a spreadsheet: a b 1 customer address 2 sally stewart 9912 school st. north wales, pa 19454 3 lorenzo price 8621 glendale dr. burlington, ma 01803 4 stella moss 372 w. addison street brandon, fl 33510 5 paul casey 9069 e. brickyard road chattanooga, tn 37421 in order to extract the five-digit postal code from north wales, pa, what is the correct function?

Answers

For the given data analysis  , The correct function is : RIGHT(B3,5), which is the correct syntax.

Here it is related to data cleaning which is the process of removing or fixing data that are incorrect or formatted incorrectly, corrupted, duplicate, or even incomplete data within a dataset that exists. When we mix  multiple data sources, it depicts that  there can be opportunities for the data to be mislabeled or duplicated, since we are guiven  that the customer names and addresses, so the correct syntax will be  =RIGHT(B3,5), where the RIGHT  Function  that returns set  number of characters of  a text string from the  right side

To know more about data analysis refer to the link brainly.com/question/23730998

#SPJ4

using an average ground speed of 120 knots and taking off on runway 14, what minimum rate of climb must be maintained to meet the required climb rate (in feet per nm) to 4,800 feet as specified on the instrument departure procedures? a. 435 feet per minute. b. 500 feet per minute. c. 1,000 feet per minute.

Answers

According to the given speed, the minimum rate of climb must be maintained to meet the required climb rate to 4,800 feet as specified on the instrument departure procedures is option (d) 870 feet per minute.

Speed:

In math speed refers the rate of change of position of an object in any direction.

Given,

Here we have using an average ground speed of 120 knots and taking off on runway 14.

Now, we need to find in what minimum rate of climb must be maintained to meet the required climb rate (in feet per nm) to 4,800 feet as specified on the instrument departure procedures.

While we looking into the given question, we have identified that,

Speed = 120

Distance = 4800

Then the  minimum rate of climb must be maintained to meet the required climb rate  is calculated as,

Here we also note in the bottom left corner of the overhead view of the departure requires 435 ft/nm climb gradient to 4800' for obstacle clearance.

Therefore, the minimum rate is 870 feet per minute.

To know more about Speed here.

https://brainly.com/question/26862717

#SPJ4

balls labeled 1 through 9 are placed in a knapsack. two balls are selected from the knapsack, one at a time without replacement. the first ball we draw is even. what is the probability that the second ball is greater than the first ball?

Answers

The probability that the second ball is greater than the first ball is 5/8

Probability is the branch of mathematics concerning numerical descriptions of how likely an event is to occur, or how likely it is that a proposition is true. The probability of an event is a number between 0 and 1, where, roughly speaking, 0 indicates impossibility of the event and 1 indicates certainty.

The ratio of the number of outcomes in an exhaustive set of equally likely outcomes that produce a given event to the total number of possible outcomes.

As to even odd

first ball can be anything. Second would have to be the first is not thus 5/9.

As second smaller than first

if first is 9, probability is 8/8, 100%, if 7 probability s 6/8 so in general take first ball, value k,

then the probability is sum (k - 1) / (n = 10 - 1) for k=1,2,…,9 so that’s all in 45/8 times 1/9 so 45/72 or 5/8.

Even a clear ball has Color so unless you have an invisible ball in your collection, both balls will be some amount of color or color.

To learn more about Probability visit:

brainly.com/question/14210034

#SPJ4

solve this please I need some help

Answers

Answer:

x = 85°

Step-by-step explanation:

the angle adjacent to B inside the triangle is corresponding to 70° and are congruent.

the sum of the 3 angles in a triangle is 180° , then

x + 25° + 70° = 180°

x + 95° = 180° ( subtract 95° from both sides )

x = 85°

How do you use demoivre's theorem to simplify (1+i)^12?

Answers

By using demovire's theorem to determine the complex numbers to simplify (1+i)^12 we get   [tex]z^{12}=-2^6=-64[/tex]

first, we have to find the argument. As the complex number, z=1+i will be in the 4th quadrant

we proceed according to conditions in demovire's theorem

[tex]r=\sqrt{(1)^2+(1)^2}\\ \\r=\sqrt{2}[/tex]

so from this our angle becomes

[tex]\alpha =2\pi -tan^{-1}=- \frac{\pi }{4}[/tex]

so putting r and alpha in the formula of demovier's of complex numbers

[tex]z^{n}=r^{n}(cos\alpha +isin\alpha )\\\\z^{12}=(\sqrt{2} )^{\112} }(cos\((\frac{-\pi }{4}) +isin(\frac{-\pi }{4}) )\\\\z^{12}=2^6(-1-0)\\\\z^{12}=-2^6=-64[/tex]

we get z to the power 12 equals to negative times sixty-four.

To learn more about demoivre's theorem, click on

brainly.com/question/29757451.

#SPJ4

Other Questions
Which expression is equivalent to 3(5.5b 3) (5b + 7)? 11.5b + 4 21.5b 2 21.5b 16 11.5b 16 The population of a city decreases by 3.3% per year. If this year's population is 72,000, what will next year's population be, to the nearest individual? for an individual cell to become a malignant line of cells, it must avoid (growth control mechanisms) for healthy cells. True/False goodness of fit 5. a buyer for a t-shirt shop wants to compare the proportion of t-shirts of each size that are sold to the proportion that were ordered. the buyer counts the number of t-shirts of each size that are sold in a week. size sold small medium 25 41 large 91 extra large 68 percentage ordered 10% 20% 40% 30% does the distribution of the sold items follow the distribution of the ordered items? state the conclusion. When a country chooses to limit the kinds of goods or services it produces, it is practicingO absolute advantage.O globalization.O dependency.O specialization. An experiment has been conducted for four treatments with eight blocks. Complete the following analysis of variance table (to 2 decimals, if necessary). Source of Variation Sum of Squares Degrees of Freedom Mean Square F Treatments 900 Blocks 400 Error Total 1800 Use A = .05 to test for any significant differences. The p-value is ? What is your conclusion? nonpolar aromatic compounds were separated by hplc on an octadecyl silyl (c18) bonded phase. the eluent was 65 vol% methanol in water. how would the retention times be affected if 90% methanol were used instead? In the passage, which line from the prom best reflects the poets point of view? what is the upper block's acceleration if the coefficient of kinetic friction between the block and the table is 0.21? 1. ______________ means that each individual has a designated supervisor to whom they report to at the scene of the incident. FILL IN THE BLANK. if obsolescence were expected to limit the longevity of a protected product, the useful life of a patent might be ___its legal life. multiple choice question. the u.s. educational system has long been based on a dominant-culture, middle-class value structure which has resulted in students from many nondominant groups being placed in remedial classes. t/f Which function has a constant additive rate of change of 1/4? A coordinate plane with a straight line with a negative slope. The line passes through (negative 2, 2) and (2, 1). A coordinate plane with a curved line passing through (negative 1, 2), (0, negative 1), the minimum (2, negative 2), and (4, negative 1). A two column table with five rows. The first column, x, has the entries, 20, 21, 22, 23. The second column, y, has the entries negative 1, negative 1.5, negative 2, negative 2.5. A two column table with five rows. The first column, x, has the entries, negative 12, negative 11, negative 10, negative 9. The second column, y, has the entries, 7, 11, 14, 17. Using Different Representations Which function has a constant additive rate of change of-1/4? y 20 -12 7 21 -1,5 -11 11 14 =2 241-2 22 ~2 -10 23 -2.5 -9 17' jim is considering pursuing an ms in information systems degree. he has applied to two different universities. the acceptance rate for applicants with similar qualifications is 30% for university x and 40% for university y. what is the probability that jim will not be accepted at either university? Describe the work johnny and katie did their first year as a married couple? assume that wheat is produced in a purely competitive market. in the short run the demand for wheat increases and wheat producers earn economic profits. in the long run how will this change in economic circumstance affect the price of wheat and the economic profits of wheat growers? price economic profits a. decrease decrease b. decrease increase c. increase decrease d. increase increase group of answer choices decrease decrease decrease increase increase decrease increase increase althea and jaime are in the process of creating a plan for their new retail store so that they can make the best use of their resources. they are meeting today to go over their ideas for the products they will sell, how they will price them, and how they will promote their new store. they are also attempting to profile the customers they will target so that they can better understand what products will appeal to them. althea and jaime are creating a Which of the following is an example of phenotype?A. 25% b. Blue c. 1:2:1 D. Bb select the word that helps the reader understand the word peer in the sentence each classmet is encouged to review work with one another during which era of federal indian policy did the following occur: tribes were thought to be sufficiently acculturated and no longer needed the protection of the u.s.; individual indian status was lost; individual indians were moved to urban areas to help complete the assimilation process